Find the values of p and q

The name of the pictureThe name of the pictureThe name of the pictureClash Royale CLAN TAG#URR8PPP











up vote
3
down vote

favorite













If $p^3+p=q^2+q$ where $p$ and $q$ are prime numbers,
Find all the solutions (p, q)




I tried to solve this exercise using that:



$p^2 = -1(textmod , q)$ and $q = -1(textmod , p)$;
So: $q+1=ap$ and $p^2+1=bq$, where $b$ and $q$ integers.



Then I tried to solve a quadratic equation, but I could not finish the problem










share|cite|improve this question



























    up vote
    3
    down vote

    favorite













    If $p^3+p=q^2+q$ where $p$ and $q$ are prime numbers,
    Find all the solutions (p, q)




    I tried to solve this exercise using that:



    $p^2 = -1(textmod , q)$ and $q = -1(textmod , p)$;
    So: $q+1=ap$ and $p^2+1=bq$, where $b$ and $q$ integers.



    Then I tried to solve a quadratic equation, but I could not finish the problem










    share|cite|improve this question

























      up vote
      3
      down vote

      favorite









      up vote
      3
      down vote

      favorite












      If $p^3+p=q^2+q$ where $p$ and $q$ are prime numbers,
      Find all the solutions (p, q)




      I tried to solve this exercise using that:



      $p^2 = -1(textmod , q)$ and $q = -1(textmod , p)$;
      So: $q+1=ap$ and $p^2+1=bq$, where $b$ and $q$ integers.



      Then I tried to solve a quadratic equation, but I could not finish the problem










      share|cite|improve this question
















      If $p^3+p=q^2+q$ where $p$ and $q$ are prime numbers,
      Find all the solutions (p, q)




      I tried to solve this exercise using that:



      $p^2 = -1(textmod , q)$ and $q = -1(textmod , p)$;
      So: $q+1=ap$ and $p^2+1=bq$, where $b$ and $q$ integers.



      Then I tried to solve a quadratic equation, but I could not finish the problem







      number-theory elementary-number-theory prime-numbers






      share|cite|improve this question















      share|cite|improve this question













      share|cite|improve this question




      share|cite|improve this question








      edited 19 mins ago

























      asked 30 mins ago









      Matheus Domingos

      856




      856




















          4 Answers
          4






          active

          oldest

          votes

















          up vote
          1
          down vote



          accepted










          $$p^3+p=q^2+q implies p|q^2+q implies p|q mathrmor p|(q+1).$$



          Obviously, $pneq q$, so $pnmid q$. Thus, we can set $q=kp-1$. This then reduces to



          $$p^2-k^2p+(k+1)=0,$$



          which has an integer solution iff $k^4-4k-4$ is a square. Can you see why this is not the case for large $k$, and determine the solutions from there?






          share|cite|improve this answer






















          • Oh thanks, now I know how to finish it, thanks
            – Matheus Domingos
            17 mins ago










          • Is there a better way to do that? I learned in high school to solve that using k^4-4k-4=a^2 and then I have to study that. Is there a better way to do that?
            – Matheus Domingos
            24 secs ago

















          up vote
          2
          down vote













          Hint: Since $p$ divides $p^3+p$, also $p$ divides $q(q+1)$, hence $p$ either divides $q$ or $p$ divides $q+1$, because $p$ is prime. Can you finish it?






          share|cite|improve this answer



























            up vote
            2
            down vote













            Clearly $pneq q$, and because $p$ and $q$ are prime and
            $$p(p^2+1)=p^3+p=q^2+q=q(q+1),$$
            we must have $pmid q+1$ and $qmid p^2+1$. Write
            $$q+1=apqquadtext and qquad p^2+1=bq,$$
            to find that $p^2-abp+b+1=0$. In particular $b+1equiv0pmodp$, say $b=cp-1$, but then
            $$p^2+1=(cp-1)q=(cp-1)(ap-1)=acp^2-(a+c)p+1.$$
            Note that $a$, $b$ and $c$ are positive integers, and that $a>1$ as otherwise $p=q+1$, so $(p,q)=(3,2)$ which is not a solution. The equation above implies that
            $$(ac-1)p=a+c,$$
            and as $pgeq2$ clearly we cannot have $a,cgeq2$. Hence $c=1$ and so
            $$p^2+1=bq=(cp-1)q=(p-1)q.$$
            In particular $p-1mid p^2+1$. As $p-1mid p^2-1$ it follows that $p-1=2$, so $p=3$ and hence $q=5$.






            share|cite|improve this answer






















            • How do I finish this solution? Finding all the solution?
              – Matheus Domingos
              8 mins ago










            • @MatheusDomingos I completed the solution for you.
              – Servaes
              7 mins ago






            • 1




              Thank you, now I understood the limitation. I need to have a+c=>ac-1... What implies that (a-1)(c-1)=>2. Thanks
              – Matheus Domingos
              1 min ago










            • I rearranged the argument a bit, as in fact $a>1$ follows very easily earlier on, simpifying the final step.
              – Servaes
              35 secs ago


















            up vote
            2
            down vote













            Remember that if $a,b$ are positive integers such that $amid b$ then $aleq b$. I'll be using this frekvently here.



            From $$p(p^2+1)= q(q+1)implies pmid q;;;rm or;;;pmid q+1$$



            1. case $pmid q$, then $q+1mid p^2+1$. Write $q+1=s$ then we get $$psmid (p^2+1)(s-1) = p^2s-p^2+s-1implies psmid p^2-s+1$$



            Since $p^2+1geq s$ we have 2 subcases:



            1.1 case $p^2+1>s$, then $psleq p^2-s+1$ so $s(p+1)leq p^2+1$, and thus $$sleq p^2+1over p+1 <pimplies sleq p-1$$



            So $q+1leq p-1 leq q-1$ and thus no solution.



            1.2 case $p^2+1=s$, then $q^2+1 = q+1$ and again no solution.



            2. case $pmid q+1$, then $qmid p^2+1$. Then we get $$pqmid (p^2+1)(q+1) = p^2q+p^2+q+1implies pqmid p^2+q+1$$



            so we have $pqleq p^2+q+1$ so $q leq p^2+1over p-1 leq p+2$ if $pgeq 3$.
            So if $pgeq 3$ and since $pmid q$ that $qin p,p+1,p+2$ which is easy to finish by hand.






            share|cite|improve this answer






















              Your Answer




              StackExchange.ifUsing("editor", function ()
              return StackExchange.using("mathjaxEditing", function ()
              StackExchange.MarkdownEditor.creationCallbacks.add(function (editor, postfix)
              StackExchange.mathjaxEditing.prepareWmdForMathJax(editor, postfix, [["$", "$"], ["\\(","\\)"]]);
              );
              );
              , "mathjax-editing");

              StackExchange.ready(function()
              var channelOptions =
              tags: "".split(" "),
              id: "69"
              ;
              initTagRenderer("".split(" "), "".split(" "), channelOptions);

              StackExchange.using("externalEditor", function()
              // Have to fire editor after snippets, if snippets enabled
              if (StackExchange.settings.snippets.snippetsEnabled)
              StackExchange.using("snippets", function()
              createEditor();
              );

              else
              createEditor();

              );

              function createEditor()
              StackExchange.prepareEditor(
              heartbeatType: 'answer',
              convertImagesToLinks: true,
              noModals: false,
              showLowRepImageUploadWarning: true,
              reputationToPostImages: 10,
              bindNavPrevention: true,
              postfix: "",
              noCode: true, onDemand: true,
              discardSelector: ".discard-answer"
              ,immediatelyShowMarkdownHelp:true
              );



              );













               

              draft saved


              draft discarded


















              StackExchange.ready(
              function ()
              StackExchange.openid.initPostLogin('.new-post-login', 'https%3a%2f%2fmath.stackexchange.com%2fquestions%2f2944795%2ffind-the-values-of-p-and-q%23new-answer', 'question_page');

              );

              Post as a guest






























              4 Answers
              4






              active

              oldest

              votes








              4 Answers
              4






              active

              oldest

              votes









              active

              oldest

              votes






              active

              oldest

              votes








              up vote
              1
              down vote



              accepted










              $$p^3+p=q^2+q implies p|q^2+q implies p|q mathrmor p|(q+1).$$



              Obviously, $pneq q$, so $pnmid q$. Thus, we can set $q=kp-1$. This then reduces to



              $$p^2-k^2p+(k+1)=0,$$



              which has an integer solution iff $k^4-4k-4$ is a square. Can you see why this is not the case for large $k$, and determine the solutions from there?






              share|cite|improve this answer






















              • Oh thanks, now I know how to finish it, thanks
                – Matheus Domingos
                17 mins ago










              • Is there a better way to do that? I learned in high school to solve that using k^4-4k-4=a^2 and then I have to study that. Is there a better way to do that?
                – Matheus Domingos
                24 secs ago














              up vote
              1
              down vote



              accepted










              $$p^3+p=q^2+q implies p|q^2+q implies p|q mathrmor p|(q+1).$$



              Obviously, $pneq q$, so $pnmid q$. Thus, we can set $q=kp-1$. This then reduces to



              $$p^2-k^2p+(k+1)=0,$$



              which has an integer solution iff $k^4-4k-4$ is a square. Can you see why this is not the case for large $k$, and determine the solutions from there?






              share|cite|improve this answer






















              • Oh thanks, now I know how to finish it, thanks
                – Matheus Domingos
                17 mins ago










              • Is there a better way to do that? I learned in high school to solve that using k^4-4k-4=a^2 and then I have to study that. Is there a better way to do that?
                – Matheus Domingos
                24 secs ago












              up vote
              1
              down vote



              accepted







              up vote
              1
              down vote



              accepted






              $$p^3+p=q^2+q implies p|q^2+q implies p|q mathrmor p|(q+1).$$



              Obviously, $pneq q$, so $pnmid q$. Thus, we can set $q=kp-1$. This then reduces to



              $$p^2-k^2p+(k+1)=0,$$



              which has an integer solution iff $k^4-4k-4$ is a square. Can you see why this is not the case for large $k$, and determine the solutions from there?






              share|cite|improve this answer














              $$p^3+p=q^2+q implies p|q^2+q implies p|q mathrmor p|(q+1).$$



              Obviously, $pneq q$, so $pnmid q$. Thus, we can set $q=kp-1$. This then reduces to



              $$p^2-k^2p+(k+1)=0,$$



              which has an integer solution iff $k^4-4k-4$ is a square. Can you see why this is not the case for large $k$, and determine the solutions from there?







              share|cite|improve this answer














              share|cite|improve this answer



              share|cite|improve this answer








              edited 12 mins ago

























              answered 19 mins ago









              Carl Schildkraut

              8,98711238




              8,98711238











              • Oh thanks, now I know how to finish it, thanks
                – Matheus Domingos
                17 mins ago










              • Is there a better way to do that? I learned in high school to solve that using k^4-4k-4=a^2 and then I have to study that. Is there a better way to do that?
                – Matheus Domingos
                24 secs ago
















              • Oh thanks, now I know how to finish it, thanks
                – Matheus Domingos
                17 mins ago










              • Is there a better way to do that? I learned in high school to solve that using k^4-4k-4=a^2 and then I have to study that. Is there a better way to do that?
                – Matheus Domingos
                24 secs ago















              Oh thanks, now I know how to finish it, thanks
              – Matheus Domingos
              17 mins ago




              Oh thanks, now I know how to finish it, thanks
              – Matheus Domingos
              17 mins ago












              Is there a better way to do that? I learned in high school to solve that using k^4-4k-4=a^2 and then I have to study that. Is there a better way to do that?
              – Matheus Domingos
              24 secs ago




              Is there a better way to do that? I learned in high school to solve that using k^4-4k-4=a^2 and then I have to study that. Is there a better way to do that?
              – Matheus Domingos
              24 secs ago










              up vote
              2
              down vote













              Hint: Since $p$ divides $p^3+p$, also $p$ divides $q(q+1)$, hence $p$ either divides $q$ or $p$ divides $q+1$, because $p$ is prime. Can you finish it?






              share|cite|improve this answer
























                up vote
                2
                down vote













                Hint: Since $p$ divides $p^3+p$, also $p$ divides $q(q+1)$, hence $p$ either divides $q$ or $p$ divides $q+1$, because $p$ is prime. Can you finish it?






                share|cite|improve this answer






















                  up vote
                  2
                  down vote










                  up vote
                  2
                  down vote









                  Hint: Since $p$ divides $p^3+p$, also $p$ divides $q(q+1)$, hence $p$ either divides $q$ or $p$ divides $q+1$, because $p$ is prime. Can you finish it?






                  share|cite|improve this answer












                  Hint: Since $p$ divides $p^3+p$, also $p$ divides $q(q+1)$, hence $p$ either divides $q$ or $p$ divides $q+1$, because $p$ is prime. Can you finish it?







                  share|cite|improve this answer












                  share|cite|improve this answer



                  share|cite|improve this answer










                  answered 20 mins ago









                  Dietrich Burde

                  75.7k64185




                  75.7k64185




















                      up vote
                      2
                      down vote













                      Clearly $pneq q$, and because $p$ and $q$ are prime and
                      $$p(p^2+1)=p^3+p=q^2+q=q(q+1),$$
                      we must have $pmid q+1$ and $qmid p^2+1$. Write
                      $$q+1=apqquadtext and qquad p^2+1=bq,$$
                      to find that $p^2-abp+b+1=0$. In particular $b+1equiv0pmodp$, say $b=cp-1$, but then
                      $$p^2+1=(cp-1)q=(cp-1)(ap-1)=acp^2-(a+c)p+1.$$
                      Note that $a$, $b$ and $c$ are positive integers, and that $a>1$ as otherwise $p=q+1$, so $(p,q)=(3,2)$ which is not a solution. The equation above implies that
                      $$(ac-1)p=a+c,$$
                      and as $pgeq2$ clearly we cannot have $a,cgeq2$. Hence $c=1$ and so
                      $$p^2+1=bq=(cp-1)q=(p-1)q.$$
                      In particular $p-1mid p^2+1$. As $p-1mid p^2-1$ it follows that $p-1=2$, so $p=3$ and hence $q=5$.






                      share|cite|improve this answer






















                      • How do I finish this solution? Finding all the solution?
                        – Matheus Domingos
                        8 mins ago










                      • @MatheusDomingos I completed the solution for you.
                        – Servaes
                        7 mins ago






                      • 1




                        Thank you, now I understood the limitation. I need to have a+c=>ac-1... What implies that (a-1)(c-1)=>2. Thanks
                        – Matheus Domingos
                        1 min ago










                      • I rearranged the argument a bit, as in fact $a>1$ follows very easily earlier on, simpifying the final step.
                        – Servaes
                        35 secs ago















                      up vote
                      2
                      down vote













                      Clearly $pneq q$, and because $p$ and $q$ are prime and
                      $$p(p^2+1)=p^3+p=q^2+q=q(q+1),$$
                      we must have $pmid q+1$ and $qmid p^2+1$. Write
                      $$q+1=apqquadtext and qquad p^2+1=bq,$$
                      to find that $p^2-abp+b+1=0$. In particular $b+1equiv0pmodp$, say $b=cp-1$, but then
                      $$p^2+1=(cp-1)q=(cp-1)(ap-1)=acp^2-(a+c)p+1.$$
                      Note that $a$, $b$ and $c$ are positive integers, and that $a>1$ as otherwise $p=q+1$, so $(p,q)=(3,2)$ which is not a solution. The equation above implies that
                      $$(ac-1)p=a+c,$$
                      and as $pgeq2$ clearly we cannot have $a,cgeq2$. Hence $c=1$ and so
                      $$p^2+1=bq=(cp-1)q=(p-1)q.$$
                      In particular $p-1mid p^2+1$. As $p-1mid p^2-1$ it follows that $p-1=2$, so $p=3$ and hence $q=5$.






                      share|cite|improve this answer






















                      • How do I finish this solution? Finding all the solution?
                        – Matheus Domingos
                        8 mins ago










                      • @MatheusDomingos I completed the solution for you.
                        – Servaes
                        7 mins ago






                      • 1




                        Thank you, now I understood the limitation. I need to have a+c=>ac-1... What implies that (a-1)(c-1)=>2. Thanks
                        – Matheus Domingos
                        1 min ago










                      • I rearranged the argument a bit, as in fact $a>1$ follows very easily earlier on, simpifying the final step.
                        – Servaes
                        35 secs ago













                      up vote
                      2
                      down vote










                      up vote
                      2
                      down vote









                      Clearly $pneq q$, and because $p$ and $q$ are prime and
                      $$p(p^2+1)=p^3+p=q^2+q=q(q+1),$$
                      we must have $pmid q+1$ and $qmid p^2+1$. Write
                      $$q+1=apqquadtext and qquad p^2+1=bq,$$
                      to find that $p^2-abp+b+1=0$. In particular $b+1equiv0pmodp$, say $b=cp-1$, but then
                      $$p^2+1=(cp-1)q=(cp-1)(ap-1)=acp^2-(a+c)p+1.$$
                      Note that $a$, $b$ and $c$ are positive integers, and that $a>1$ as otherwise $p=q+1$, so $(p,q)=(3,2)$ which is not a solution. The equation above implies that
                      $$(ac-1)p=a+c,$$
                      and as $pgeq2$ clearly we cannot have $a,cgeq2$. Hence $c=1$ and so
                      $$p^2+1=bq=(cp-1)q=(p-1)q.$$
                      In particular $p-1mid p^2+1$. As $p-1mid p^2-1$ it follows that $p-1=2$, so $p=3$ and hence $q=5$.






                      share|cite|improve this answer














                      Clearly $pneq q$, and because $p$ and $q$ are prime and
                      $$p(p^2+1)=p^3+p=q^2+q=q(q+1),$$
                      we must have $pmid q+1$ and $qmid p^2+1$. Write
                      $$q+1=apqquadtext and qquad p^2+1=bq,$$
                      to find that $p^2-abp+b+1=0$. In particular $b+1equiv0pmodp$, say $b=cp-1$, but then
                      $$p^2+1=(cp-1)q=(cp-1)(ap-1)=acp^2-(a+c)p+1.$$
                      Note that $a$, $b$ and $c$ are positive integers, and that $a>1$ as otherwise $p=q+1$, so $(p,q)=(3,2)$ which is not a solution. The equation above implies that
                      $$(ac-1)p=a+c,$$
                      and as $pgeq2$ clearly we cannot have $a,cgeq2$. Hence $c=1$ and so
                      $$p^2+1=bq=(cp-1)q=(p-1)q.$$
                      In particular $p-1mid p^2+1$. As $p-1mid p^2-1$ it follows that $p-1=2$, so $p=3$ and hence $q=5$.







                      share|cite|improve this answer














                      share|cite|improve this answer



                      share|cite|improve this answer








                      edited 7 mins ago

























                      answered 20 mins ago









                      Servaes

                      18.8k33684




                      18.8k33684











                      • How do I finish this solution? Finding all the solution?
                        – Matheus Domingos
                        8 mins ago










                      • @MatheusDomingos I completed the solution for you.
                        – Servaes
                        7 mins ago






                      • 1




                        Thank you, now I understood the limitation. I need to have a+c=>ac-1... What implies that (a-1)(c-1)=>2. Thanks
                        – Matheus Domingos
                        1 min ago










                      • I rearranged the argument a bit, as in fact $a>1$ follows very easily earlier on, simpifying the final step.
                        – Servaes
                        35 secs ago

















                      • How do I finish this solution? Finding all the solution?
                        – Matheus Domingos
                        8 mins ago










                      • @MatheusDomingos I completed the solution for you.
                        – Servaes
                        7 mins ago






                      • 1




                        Thank you, now I understood the limitation. I need to have a+c=>ac-1... What implies that (a-1)(c-1)=>2. Thanks
                        – Matheus Domingos
                        1 min ago










                      • I rearranged the argument a bit, as in fact $a>1$ follows very easily earlier on, simpifying the final step.
                        – Servaes
                        35 secs ago
















                      How do I finish this solution? Finding all the solution?
                      – Matheus Domingos
                      8 mins ago




                      How do I finish this solution? Finding all the solution?
                      – Matheus Domingos
                      8 mins ago












                      @MatheusDomingos I completed the solution for you.
                      – Servaes
                      7 mins ago




                      @MatheusDomingos I completed the solution for you.
                      – Servaes
                      7 mins ago




                      1




                      1




                      Thank you, now I understood the limitation. I need to have a+c=>ac-1... What implies that (a-1)(c-1)=>2. Thanks
                      – Matheus Domingos
                      1 min ago




                      Thank you, now I understood the limitation. I need to have a+c=>ac-1... What implies that (a-1)(c-1)=>2. Thanks
                      – Matheus Domingos
                      1 min ago












                      I rearranged the argument a bit, as in fact $a>1$ follows very easily earlier on, simpifying the final step.
                      – Servaes
                      35 secs ago





                      I rearranged the argument a bit, as in fact $a>1$ follows very easily earlier on, simpifying the final step.
                      – Servaes
                      35 secs ago











                      up vote
                      2
                      down vote













                      Remember that if $a,b$ are positive integers such that $amid b$ then $aleq b$. I'll be using this frekvently here.



                      From $$p(p^2+1)= q(q+1)implies pmid q;;;rm or;;;pmid q+1$$



                      1. case $pmid q$, then $q+1mid p^2+1$. Write $q+1=s$ then we get $$psmid (p^2+1)(s-1) = p^2s-p^2+s-1implies psmid p^2-s+1$$



                      Since $p^2+1geq s$ we have 2 subcases:



                      1.1 case $p^2+1>s$, then $psleq p^2-s+1$ so $s(p+1)leq p^2+1$, and thus $$sleq p^2+1over p+1 <pimplies sleq p-1$$



                      So $q+1leq p-1 leq q-1$ and thus no solution.



                      1.2 case $p^2+1=s$, then $q^2+1 = q+1$ and again no solution.



                      2. case $pmid q+1$, then $qmid p^2+1$. Then we get $$pqmid (p^2+1)(q+1) = p^2q+p^2+q+1implies pqmid p^2+q+1$$



                      so we have $pqleq p^2+q+1$ so $q leq p^2+1over p-1 leq p+2$ if $pgeq 3$.
                      So if $pgeq 3$ and since $pmid q$ that $qin p,p+1,p+2$ which is easy to finish by hand.






                      share|cite|improve this answer


























                        up vote
                        2
                        down vote













                        Remember that if $a,b$ are positive integers such that $amid b$ then $aleq b$. I'll be using this frekvently here.



                        From $$p(p^2+1)= q(q+1)implies pmid q;;;rm or;;;pmid q+1$$



                        1. case $pmid q$, then $q+1mid p^2+1$. Write $q+1=s$ then we get $$psmid (p^2+1)(s-1) = p^2s-p^2+s-1implies psmid p^2-s+1$$



                        Since $p^2+1geq s$ we have 2 subcases:



                        1.1 case $p^2+1>s$, then $psleq p^2-s+1$ so $s(p+1)leq p^2+1$, and thus $$sleq p^2+1over p+1 <pimplies sleq p-1$$



                        So $q+1leq p-1 leq q-1$ and thus no solution.



                        1.2 case $p^2+1=s$, then $q^2+1 = q+1$ and again no solution.



                        2. case $pmid q+1$, then $qmid p^2+1$. Then we get $$pqmid (p^2+1)(q+1) = p^2q+p^2+q+1implies pqmid p^2+q+1$$



                        so we have $pqleq p^2+q+1$ so $q leq p^2+1over p-1 leq p+2$ if $pgeq 3$.
                        So if $pgeq 3$ and since $pmid q$ that $qin p,p+1,p+2$ which is easy to finish by hand.






                        share|cite|improve this answer
























                          up vote
                          2
                          down vote










                          up vote
                          2
                          down vote









                          Remember that if $a,b$ are positive integers such that $amid b$ then $aleq b$. I'll be using this frekvently here.



                          From $$p(p^2+1)= q(q+1)implies pmid q;;;rm or;;;pmid q+1$$



                          1. case $pmid q$, then $q+1mid p^2+1$. Write $q+1=s$ then we get $$psmid (p^2+1)(s-1) = p^2s-p^2+s-1implies psmid p^2-s+1$$



                          Since $p^2+1geq s$ we have 2 subcases:



                          1.1 case $p^2+1>s$, then $psleq p^2-s+1$ so $s(p+1)leq p^2+1$, and thus $$sleq p^2+1over p+1 <pimplies sleq p-1$$



                          So $q+1leq p-1 leq q-1$ and thus no solution.



                          1.2 case $p^2+1=s$, then $q^2+1 = q+1$ and again no solution.



                          2. case $pmid q+1$, then $qmid p^2+1$. Then we get $$pqmid (p^2+1)(q+1) = p^2q+p^2+q+1implies pqmid p^2+q+1$$



                          so we have $pqleq p^2+q+1$ so $q leq p^2+1over p-1 leq p+2$ if $pgeq 3$.
                          So if $pgeq 3$ and since $pmid q$ that $qin p,p+1,p+2$ which is easy to finish by hand.






                          share|cite|improve this answer














                          Remember that if $a,b$ are positive integers such that $amid b$ then $aleq b$. I'll be using this frekvently here.



                          From $$p(p^2+1)= q(q+1)implies pmid q;;;rm or;;;pmid q+1$$



                          1. case $pmid q$, then $q+1mid p^2+1$. Write $q+1=s$ then we get $$psmid (p^2+1)(s-1) = p^2s-p^2+s-1implies psmid p^2-s+1$$



                          Since $p^2+1geq s$ we have 2 subcases:



                          1.1 case $p^2+1>s$, then $psleq p^2-s+1$ so $s(p+1)leq p^2+1$, and thus $$sleq p^2+1over p+1 <pimplies sleq p-1$$



                          So $q+1leq p-1 leq q-1$ and thus no solution.



                          1.2 case $p^2+1=s$, then $q^2+1 = q+1$ and again no solution.



                          2. case $pmid q+1$, then $qmid p^2+1$. Then we get $$pqmid (p^2+1)(q+1) = p^2q+p^2+q+1implies pqmid p^2+q+1$$



                          so we have $pqleq p^2+q+1$ so $q leq p^2+1over p-1 leq p+2$ if $pgeq 3$.
                          So if $pgeq 3$ and since $pmid q$ that $qin p,p+1,p+2$ which is easy to finish by hand.







                          share|cite|improve this answer














                          share|cite|improve this answer



                          share|cite|improve this answer








                          edited 2 mins ago

























                          answered 18 mins ago









                          greedoid

                          30.7k94184




                          30.7k94184



























                               

                              draft saved


                              draft discarded















































                               


                              draft saved


                              draft discarded














                              StackExchange.ready(
                              function ()
                              StackExchange.openid.initPostLogin('.new-post-login', 'https%3a%2f%2fmath.stackexchange.com%2fquestions%2f2944795%2ffind-the-values-of-p-and-q%23new-answer', 'question_page');

                              );

                              Post as a guest













































































                              Comments

                              Popular posts from this blog

                              Is the Concept of Multiple Fantasy Races Scientifically Flawed? [closed]

                              Long meetings (6-7 hours a day): Being “babysat” by supervisor

                              Confectionery